Circled Part Formula in Double Integral: Explaining the Use of dA in Polar Form?

Click For Summary
The discussion centers on understanding the conversion of the area element dA in polar coordinates, specifically why dA becomes r(dr)(dθ). The formula dA = r ⋅ dr ⋅ dθ represents the area of an infinitesimal ring with radius r and thickness dr, not the total area of a circle. The confusion arises from the integration process, where 2π appears only when integrating over θ for functions that do not depend on θ. Thus, the integral can be expressed as a product of the radial and angular components, leading to the inclusion of 2π only under specific conditions. This clarification helps in understanding the use of dA in double integrals in polar form.
chetzread
Messages
798
Reaction score
1

Homework Statement


can someone explain about the formula of the circled part?
Why dA will become r(dr)(dθ)?
MhTV4wv.png

Homework Equations

The Attempt at a Solution


A = pi(r^2)
dA will become 2(pi)(r)(dr) ?
why did 2(pi) didnt appear in the equation ?
 
Physics news on Phys.org
chetzread said:

Homework Statement


can someone explain about the formula of the circled part?
Why dA will become r(dr)(dθ)?
MhTV4wv.png

Homework Equations

The Attempt at a Solution


A = pi(r^2)
dA will become 2(pi)(r)(dr) ?
why did 2(pi) didnt appear in the equation ?
dA = r ⋅ dr ⋅ dθ as can be seen from the diagram below:


polar_coordinates_area_calculation.png


2π has nothing to do with converting dA into its polar equivalent.
 
  • Like
Likes chetzread
chetzread said:
A = pi(r^2)
dA will become 2(pi)(r)(dr)?
The dA you calculated here is the infinitesimal increase in the area of a circle if you increase the radius by dr. In other words, it's the area of a ring of radius r and width dr. It's not the same dA that appears in the integral.
 
  • Like
Likes chetzread
chetzread said:
why did 2(pi) didnt appear in the equation ?
Is it clear to you now that this hasn't happened because the integral reads$$
\displaystyle \iint\limits_R f(x,y)\; dA = \iint\limits_R f(r,\theta)\; r \;dr\;d\theta\quad ?$$Only if ##\ f(r,\theta) = f(r)\ ## i.e. f does not depend on ##\theta##, the integration over ##d\theta## can be carried out (yielding ##2\pi##) and the result is$$
\displaystyle \iint\limits_R f(x,y)\; dA = \int\limits_{r_0}^{r_1} f(r)\; r \;dr\;\int\limits_0^{2\pi} d\theta\quad = 2\pi \int\limits_{r_0}^{r_1} f(r)\; r \;dr\;$$
 
BvU said:
Is it clear to you now that this hasn't happened because the integral reads$$
\displaystyle \iint\limits_R f(x,y)\; dA = \iint\limits_R f(r,\theta)\; r \;dr\;d\theta\quad ?$$Only if ##\ f(r,\theta) = f(r)\ ## i.e. f does not depend on ##\theta##, the integration over ##d\theta## can be carried out (yielding ##2\pi##) and the result is$$
\displaystyle \iint\limits_R f(x,y)\; dA = \int\limits_{r_0}^{r_1} f(r)\; r \;dr\;\int\limits_0^{2\pi} d\theta\quad = 2\pi \int\limits_{r_0}^{r_1} f(r)\; r \;dr\;$$
yes, thanks
 
Question: A clock's minute hand has length 4 and its hour hand has length 3. What is the distance between the tips at the moment when it is increasing most rapidly?(Putnam Exam Question) Answer: Making assumption that both the hands moves at constant angular velocities, the answer is ## \sqrt{7} .## But don't you think this assumption is somewhat doubtful and wrong?

Similar threads

  • · Replies 19 ·
Replies
19
Views
2K
Replies
3
Views
2K
  • · Replies 3 ·
Replies
3
Views
1K
  • · Replies 12 ·
Replies
12
Views
2K
  • · Replies 3 ·
Replies
3
Views
3K
  • · Replies 3 ·
Replies
3
Views
2K
  • · Replies 13 ·
Replies
13
Views
2K
Replies
6
Views
2K
  • · Replies 14 ·
Replies
14
Views
3K
Replies
5
Views
2K